Difference between revisions of "1980 USAMO Problems/Problem 1"

m (Solution)
 
(14 intermediate revisions by 3 users not shown)
Line 1: Line 1:
== Problem ==
 
A balance has unequal arms and pans of unequal weight. It is used to weigh three objects. The first object balances against a weight <math>A</math>, when placed in the left pan and against a weight <math>a</math>, when placed in the right pan. The corresponding weights for the second object are <math>B</math> and <math>b</math>. The third object balances against a weight <math>C</math>, when placed in the left pan. What is its true weight?
 
 
 
== Solution ==
 
== Solution ==
  
A balance scale will balance when the torques exerted on both sides cancel out.  On each of the two sides, the total torque will be <math>\text{[some constant amount] (due to the weight, and distribution of the weight, of the arm itself) } + \text{ [the length of the arm] } \times \text{ [the weight of what is sitting in the pan]}</math>.  Thus, the information we have tells us that, for some constants <math>x, y, z, u</math>:
+
The effect of the unequal arms and pans is that if an object of weight <math> x</math> in the left pan balances an object of weight <math>y </math> in the right pan, then <math> x = hy + k</math> for some constants <math>h</math> and <math>k</math>. Thus if the first object has true weight x, then <math> x = hA + k, a = hx + k</math>.
 
 
<cmath>x + yA = z + ua</cmath>
 
<cmath>x + yB = z + ub</cmath>
 
<cmath>x + yC = z + uc</cmath>
 
 
 
In fact, we don't exactly care what <math>x,y,z,u</math> are.  By subtracting <math>x</math> from all equations and dividing by <math>y</math>, we get:
 
 
 
<cmath>A = \frac{z-x}{y} + a\left(\frac{u}{y}\right)</cmath>
 
<cmath>B = \frac{z-x}{y} + b\left(\frac{u}{y}\right)</cmath>
 
<cmath>C = \frac{z-x}{y} + c\left(\frac{u}{y}\right)</cmath>
 
  
We can just give the names <math>X</math> and <math>Y</math> to the quantities <math>\frac{z-x}{y}</math> and <math>\frac{u}{y}</math>.
+
So <math>a = h^2A + (h+1)k</math>.  
  
<cmath>A = X + Ya</cmath>
+
Similarly, <math>b = h^2B + (h+1)k</math>. Subtracting gives <math>h^2 = \frac{a-b}{A-B} </math> and so
<cmath>B = X + Yb</cmath>
 
<cmath>C = X + Yc</cmath>
 
  
Our task is to compute <math>c</math> in terms of <math>A</math>, <math>a</math>, <math>B</math>, <math>b</math>, and <math>C</math>.  This can be done by solving for <math>X</math> and <math>Y</math> in terms of <math>A</math>,<math>a</math>,<math>B</math>,<math>b</math> and eliminating them from the implicit expression for <math>c</math> in the last equation. Perhaps there is a shortcut, but this will work:
+
<cmath>(h+1)k = a - h^2A = \frac{bA - aB}{A - B}</cmath>.
  
<cmath>A = X + Ya\implies \boxed{X = A - Ya}</cmath>
+
The true weight of the third object is thus:
<cmath>B = X + Yb\implies B = A - Ya + Yb\implies Y(b-a) = B-A\implies Y = \frac{B-A}{b-a}\implies X = \boxed{A - a\left(\frac{B-A}{b-a}\right)}</cmath>
 
<cmath>C = X + Yc\implies Yc = C - X\implies c = \frac{C-X}{Y}\implies c = \frac{C - [A - \frac{B-A}{b-a} \cdot a]}{\frac{B-A}{b-a}}
 
\implies c = \frac{C - A + a\left(\frac{B-A}{b-a}\right)}{\frac{B-A}{b-a}}
 
\implies c = \frac{C(b-a) - A(b-a) + a(B-A)}{B-A}
 
\implies c = \frac{Cb - Ca - Ab + Aa + Ba - Aa}{B-A}
 
\implies  c = \frac{Cb - Ca - Ab + Ba}{B-A}</cmath>
 
  
So the answer is: <cmath>\boxed{\frac{Cb - Ca - Ab + Ba}{B-A}}</cmath>.
+
<cmath>
 +
hC + k = \\
 +
\boxed{\sqrt{ \frac{a-b}{A-B}} C + \frac{bA - aB}{(A - B)(\sqrt{ \frac {a-b}{A-B}}+ 1)}}
 +
</cmath>.
  
== See Also ==
+
More readably:
{{USAMO box|year=1980|before=First Question|num-a=2}}
+
<cmath>
 +
\boxed{ h=\sqrt{\frac{a-b}{A-B}} ;
 +
\\
 +
\text{weight} = hC + \frac{bA - aB}{(A - B)(h + 1)}}
 +
</cmath>
  
[[Category:Olympiad Algebra Problems]]
+
Credit: John Scholes https://prase.cz/kalva/usa/usoln/usol801.html

Latest revision as of 14:46, 26 March 2023

Solution

The effect of the unequal arms and pans is that if an object of weight $x$ in the left pan balances an object of weight $y$ in the right pan, then $x = hy + k$ for some constants $h$ and $k$. Thus if the first object has true weight x, then $x = hA + k, a = hx +  k$.

So $a = h^2A + (h+1)k$.

Similarly, $b = h^2B + (h+1)k$. Subtracting gives $h^2 = \frac{a-b}{A-B}$ and so

\[(h+1)k = a - h^2A = \frac{bA - aB}{A - B}\].

The true weight of the third object is thus:

\[hC + k = \\ \boxed{\sqrt{ \frac{a-b}{A-B}} C + \frac{bA - aB}{(A - B)(\sqrt{ \frac {a-b}{A-B}}+ 1)}}\].

More readably: \[\boxed{ h=\sqrt{\frac{a-b}{A-B}} ;  \\ \text{weight} = hC + \frac{bA - aB}{(A - B)(h + 1)}}\]

Credit: John Scholes https://prase.cz/kalva/usa/usoln/usol801.html